Difference between revisions of "2012 AMC 10B Problems/Problem 15"

(Created page with "The total amount of games in the tournament is 5+4+3+2+1=15. Now, we see which numbers from 1-6 divide 15, and it seems 1,3, and 5 do. 5 is the largest number, so (D) 5 is the c...")
 
(Solution)
(32 intermediate revisions by 16 users not shown)
Line 1: Line 1:
The total amount of games in the tournament is 5+4+3+2+1=15.  
+
==Problem==
Now, we see which numbers from 1-6 divide 15, and it seems 1,3, and 5 do. 5 is the largest number, so (D) 5 is the correct answer.
+
In a round-robin tournament with 6 teams, each team plays one game against each other team, and each game results in one team winning and one team losing. At the end of the tournament, the teams are ranked by the number of games won. What is the maximum number of teams that could be tied for the most wins at the end on the tournament?
 +
 
 +
<math> \textbf{(A)}\ 2\qquad\textbf{(B)}\ 3\qquad\textbf{(C)}\ 4\qquad\textbf{(D)}\ 5\qquad\textbf{(E)}\ 6 </math>
 +
==Solution==
 +
The total number of games (and wins) in the tournament is <math>\frac{6 \times 5}{2}= 15</math>. A six-way tie is impossible as this would imply each team has 2.5 wins, so the maximum number of tied teams is five. Here's a chart of 15 games where five teams each have 3 wins:
 +
|  1  2  3  4  5  6 |
 +
|1 X  W  L  W  L  W |
 +
|2 L  X  W  L  W  W |
 +
|3 W  L  X  W  L  W |
 +
|4 L  W  L  X  W  W |
 +
|5 W  L  W  L  X  W |
 +
|6 L  L  L  L  L  X |
 +
The "X's" are for when it is where a team is set against itself, which cannot happen. The chart says that Team 6 has lost all of its matches, which means that each of the other teams won against it. Then, alternating Wins and Losses were tried. It shows that it is possible for 5 teams to tie for the same amount of wins, which in this case is 3 wins.
 +
Thus, the answer is <math>\boxed{\textbf{(D)}\ 5}</math>.
 +
 
 +
==Solution 2==
 +
just note that total matches are 15, hence and if 4 teams tie for most wins then they can tie for 3 wins each, but then 5th team also has 3 wins hence 4 + 1 = 5 is the answer
 +
 
 +
==See Also==
 +
 
 +
{{AMC10 box|year=2012|ab=B|num-b=14|num-a=16}}
 +
{{MAA Notice}}

Revision as of 03:30, 16 August 2020

Problem

In a round-robin tournament with 6 teams, each team plays one game against each other team, and each game results in one team winning and one team losing. At the end of the tournament, the teams are ranked by the number of games won. What is the maximum number of teams that could be tied for the most wins at the end on the tournament?

$\textbf{(A)}\ 2\qquad\textbf{(B)}\ 3\qquad\textbf{(C)}\ 4\qquad\textbf{(D)}\ 5\qquad\textbf{(E)}\ 6$

Solution

The total number of games (and wins) in the tournament is $\frac{6 \times 5}{2}= 15$. A six-way tie is impossible as this would imply each team has 2.5 wins, so the maximum number of tied teams is five. Here's a chart of 15 games where five teams each have 3 wins:

|  1  2  3  4  5  6 |
|1 X  W  L  W  L  W |
|2 L  X  W  L  W  W |
|3 W  L  X  W  L  W |
|4 L  W  L  X  W  W |
|5 W  L  W  L  X  W |
|6 L  L  L  L  L  X |

The "X's" are for when it is where a team is set against itself, which cannot happen. The chart says that Team 6 has lost all of its matches, which means that each of the other teams won against it. Then, alternating Wins and Losses were tried. It shows that it is possible for 5 teams to tie for the same amount of wins, which in this case is 3 wins. Thus, the answer is $\boxed{\textbf{(D)}\ 5}$.

Solution 2

just note that total matches are 15, hence and if 4 teams tie for most wins then they can tie for 3 wins each, but then 5th team also has 3 wins hence 4 + 1 = 5 is the answer

See Also

2012 AMC 10B (ProblemsAnswer KeyResources)
Preceded by
Problem 14
Followed by
Problem 16
1 2 3 4 5 6 7 8 9 10 11 12 13 14 15 16 17 18 19 20 21 22 23 24 25
All AMC 10 Problems and Solutions

The problems on this page are copyrighted by the Mathematical Association of America's American Mathematics Competitions. AMC logo.png